La ricerca ha trovato 633 risultati

da Anér
23 ott 2008, 20:14
Forum: Geometria
Argomento: Carica in equilibrio all'interno di un triangolo
Risposte: 6
Visite : 3215

|\overrightarrow{v_A}|=\frac{kq_A}{|\overrightarrow{A}-\overrightarrow{P}|^2} \overrightarrow{v_A}= \frac{\overrightarrow{A}-\overrightarrow{P}}{|\overrightarrow{A}-\overrightarrow{P}|}\frac{kq_A}{|\overrightarrow{A}-\overrightarrow{P}|^2}=\frac{(\overrightarrow{A}-\overrightarrow{P})kq_A}{|\overri...
da Anér
19 ott 2008, 17:25
Forum: Combinatoria
Argomento: Le Parti sono più del Tutto
Risposte: 11
Visite : 4668

La dimostrazione che ho trovato è simile a quella per dimostrare che i numeri reali sono più dei razionali. Supponiamo per assurdo che ci sia una funzione suriettiva dall'insieme A all'insieme P(A) . Allora tutti gli elementi di P(A) , ovvero tutti i sottoinsiemi di A, sono raggiunti dalla funzione....
da Anér
11 ott 2008, 22:57
Forum: Geometria
Argomento: Tre circonferenze e assi radicali
Risposte: 2
Visite : 1899

Effettivamente questo problema è più semplice di quanto mi aspettavo; io avevo avevo previsto una soluzione con più calcoli, sfruttando il fatto che la potenza di un punto P rispetto a una crf di centro O e raggio R è $ OP^2-R^2 $.
da Anér
10 ott 2008, 14:05
Forum: Matematica non elementare
Argomento: Disuguaglianza con \pi
Risposte: 17
Visite : 7054

Davide90 ha scritto:$ $ \displaystile \left( \frac{ \pi +1}{\pi} \right) ^{\pi + 1} < \pi $
$ $ \displaystile \frac { (\pi +1)^{\pi +1} }{ \pi ^{\pi +1}} <\pi $
$ $ (\pi + 1)^ {\pi + 1} < \pi ^ {\pi + 2} $
Può servire :?: :roll:
Certo, a questo punto si fanno i calcoli e la disuguaglianza risulta!
da Anér
10 ott 2008, 13:31
Forum: Geometria
Argomento: Tre circonferenze e assi radicali
Risposte: 2
Visite : 1899

Tre circonferenze e assi radicali

Siano \Gamma_A , \Gamma_B e \Gamma_C tre circonferenze di centri O_A , O_B e O_C . Si sa che O_A sta sull'asse radicale di \Gamma_B e \Gamma_C , e che O_B sta sull'asse radicale di \Gamma_A e \Gamma_C . Dimostrare che anche O_C sta sull'asse radicale di \Gamma_A e \Gamma_B . Dimostrare che se \Gamma...
da Anér
01 ott 2008, 18:21
Forum: Algebra
Argomento: Numerabilità e basi di Hamel
Risposte: 1
Visite : 1930

Numerabilità e basi di Hamel

Una base di Hamel è numerabile?
da Anér
07 set 2008, 07:22
Forum: Fisica
Argomento: Quesiti vari
Risposte: 11
Visite : 6829

Forse ho capito. Se i battimenti si sentono quando la differenza dei suoni è inferiore a 16 Hz ma non è nulla, allora al pianista basta trovare i due valori estremi dell'intervallo (tendendo più o meno la corda), e poi fare la media delle tensioni; poiché la frequenza è direttamente proporzionale al...
da Anér
06 set 2008, 18:49
Forum: Fisica
Argomento: Quesiti vari
Risposte: 11
Visite : 6829

ico1989 ha scritto: Penso $ f_{La} = f_{diapason} $
Scusami, ma continuo a non capire; la frequenza del diapason è 440 Hz, in quanto battendo il diapason esso da il la, quindi per ottenere il la basta battere il diapason. Ma un accordo perfatto non è l'insieme di tonica, modale e dominante di una scala?
da Anér
05 set 2008, 20:34
Forum: Fisica
Argomento: Quesiti vari
Risposte: 11
Visite : 6829

Aggiungo questa domanda sui battimenti: Supponiamo che un accordatore di pianoforti volesse accordare il La centrale del pianoforte ( 400,0 Hz) usando il diapason opportuno. Quanto tempo ci impiegherebbe, ascoltando i battimenti, se volesse ottenere un accordo perfetto? 1) Non erano 440 Hz il la ce...
da Anér
05 set 2008, 15:20
Forum: Olimpiadi della matematica
Argomento: Febbraio
Risposte: 9
Visite : 5328

giove ha scritto:Qua non è stato premiato nessuno, quindi mettiti il cuore in pace.
Qua invece neanche
da Anér
24 ago 2008, 21:08
Forum: Teoria dei Numeri
Argomento: N! |P Numeri consecutivi e divisibilità
Risposte: 7
Visite : 4244

Dimostriamo che per ogni fattore primo p di N! il prodotto da k+1 a k+N ha nella fattorizzazione un esponente maggiore o uguale a quello di N!. Sia x la massima potenza di p che divide uno dei numeri da 1 a N . Siano inoltre m_1 i multipli di p nell'insieme dei numeri da 1 a N , siano m_2 i multipli...
da Anér
23 ago 2008, 19:29
Forum: Combinatoria
Argomento: Per chi è agli inizi con i conteggi
Risposte: 15
Visite : 5974

Zero! Perché Gianni deve comprare il gelato a tutti con i soldi ricevuti! :D
da Anér
23 ago 2008, 15:39
Forum: Teoria dei Numeri
Argomento: risolvere un bel problema
Risposte: 5
Visite : 3046

Significa che, scrivendo il numero con la virgola, il periodo del numero non è b-1; in base 10 ad esempio significa che non può venire 9 periodico, il che è vero perché 9 periodico si sostituisce con 0 aggiungendo 1 all'ultima cifra prima del periodo. Se in base b avessimo b-1 periodico, allora il n...
da Anér
20 ago 2008, 10:50
Forum: Geometria
Argomento: Triangoli non più isosceli e rette concorrenti
Risposte: 1
Visite : 1717

Triangoli non più isosceli e rette concorrenti

Sia ABC un triangolo. Si costruiscano verso l'esterno (oppure verso l'interno) sui lati di ABC tre triangoli A'BC, AB'C e ABC' simili tra loro. Dimostrare che AA', BB' e CC' concorrono.
da Anér
19 ago 2008, 13:39
Forum: Ciao a tutti, mi presento:
Argomento: Ciao a tutti!
Risposte: 11
Visite : 5471

Welcome!